Lifting The Exponent Lemma - Version 4
Lifting The Exponent Lemma - Version 4
Lifting The Exponent Lemma - Version 4
| x but p
+1
x. We also write p
x, if and only
if x
p
= . So we have xy
p
= x
p
y
p
and x + y
p
min {x
p
, y
p
} .
Often the symbol v
p
is used for what we denote here by
p
and the symbol
x
p
is then used for e
vp(x)
.
Example. The greatest power of 3 that divides 63 is 3
2
. because 3
2
= 9 | 63
but 3
3
= 27 63. in particular, 3
2
63 or 63
3
= 2, and we write v
3
(63) = 2.
Example. Clearly we see that if p and q be two dierent prime numbers, then
p
p
= , or p
, and we write v
p
(p
) = .
Note. We have 0
p
= for all primes p.
1
2 Two Important and Useful Lemmas
Lemma 1. Let x and y be (not necessary positive) integers and let n be a
positive integer. Given an arbitrary prime p (e.g. we can have p = 2) such that
gcd(n, p) = 1, p | x y and neither x nor y is divisible by p (in particular, p x
and p y). We have
v
p
(x
n
y
n
) = v
p
(x y).
Proof. We use the fact that
x
n
y
n
= (x y)(x
n1
+ x
n2
y + x
n3
y
2
+ + y
n1
).
Now if we show that p x
n1
+x
n2
y +x
n3
y
2
+ +y
n1
, then we are done.
In order to show this, we use the assumption p | x y. So we have x y 0
(mod p), or x y (mod p). Thus
x
n1
+x
n2
y + x
n3
y
2
+ + y
n1
x
n1
+ x
n2
x + x
n3
x
2
+ + x x
n2
+ x
n1
nx
n1
0 (mod p).
This completes the proof.
Lemma 2. Let x and y be (not necessary positive) integers and let n be an odd
positive integer. Given an arbitrary prime p (e.g. we can have p = 2) such that
gcd(n, p) = 1, p | x + y and neither x nor y is divisible by p, we have
v
p
(x
n
+ y
n
) = v
p
(x + y).
Proof. Since x and y can be negative, using Lemma 1 we obtain
v
p
(x
n
(y)
n
) = v
p
(x (y)) = v
p
(x
n
+ y
n
) = v
p
(x + y).
Note that since n is an odd positive integer we can replace (y)
n
with y
n
.
3 Lifting The Exponent Lemma (LTE)
Theorem 1 (First Form of LTE). Let x and y be (not necessary positive)
integers and let n be a positive integer and p be an odd prime such that p | xy
and none of x and y are divisible by p (in particular, p x and p y). We have
v
p
(x
n
y
n
) = v
p
(x y) + v
p
(n).
Proof. We may use induction on v
p
(n). First, let us prove the following state-
ment:
v
p
(x
p
y
p
) = v
p
(x y) + 1. (1)
2
In order to prove this, we will show that
p | x
p1
+ x
p2
y + + xy
p2
+ y
p1
(2)
and
p
2
x
p1
+ x
p2
y + + xy
p2
+ y
p1
. (3)
For (2), we note that
x
p1
+ x
p2
y + + xy
p2
+ y
p1
px
p1
0 (mod p).
Now, let y = x+kp, where k is an integer. For an integer 1 t < p we have
y
t
x
p1t
(x + kp)
t
x
p1t
x
p1t
_
x
t
+ t(kp)(x
t1
) +
t(t 1)
2
(kp)
2
(x
t2
) +
_
x
p1t
_
x
t
+ t(kp)(x
t1
)
_
x
p1
+ tkpx
p2
(mod p
2
).
This means
y
t
x
p1t
x
p1
+ tkpx
p2
(mod p
2
), t = 1, 2, 3, 4, . . . , p 1.
Using this fact, we have
x
p1
+ x
p2
y + + xy
p2
+ y
p1
x
p1
+ (x
p1
+ kpx
p2
) + (x
p1
+ 2kpx
p2
) + + (x
p1
+ (p 1)kpx
p2
)
px
p1
+ (1 + 2 + + p 1)kpx
p2
px
p1
+
_
p(p 1)
2
_
kpx
p2
px
p1
+
_
p 1
2
_
kp
2
x
p1
px
p1
0 (mod p
2
).
So we proved (3) and the proof of (1) is complete. Now let us return to our
problem. We want to show that
v
p
(x
n
y
n
) = v
p
(x y) + v
p
(n).
Suppose that n = p
b where gcd(p, b) = 1. So
x
n
y
n
p
= (x
p
)
b
(y
p
)
b
p
= x
p
y
p
p
= (x
p
1
)
p
(y
p
1
)
p
p
= x
p
1
y
p
1
p
+ 1 = (x
p
2
)
p
(y
p
2
)
p
p
+ 1
= x
p
2
y
p
2
p
+ 2
.
.
.
= (x
p
1
)
1
(y
p
1
)
1
p
+ 1 = x y
p
+
= x y
p
+n
p
.
3
Note that we used the fact that if p | xy, then we have p | x
k
y
k
, because
we have x y | x
k
y
k
for all positive integers k. The proof is complete.
Theorem 2 (Second Form of LTE). Let x, y be two integers, n be an odd
positive integer, and p be an odd prime such that p | x +y and none of x and y
are divisible by p. We have
v
p
(x
n
+ y
n
) = v
p
(x + y) + v
p
(n).
Proof. This is obvious using Theorem 1. See the trick we used in proof of
Lemma 2.
4 What about p = 2?
Question. Why did we assume that p is an odd prime, in particular,, p = 2?
Why cant we assume that p = 2 in our proofs?
Hint. Note that
p1
2
is an integer only for p > 2.
Theorem 3 (LTE for the case p = 2). Let x and y be two odd integers such
that 4 | x y. Then
v
2
(x
n
y
n
) = v
2
(x y) + v
2
(n).
Proof. We showed that for any prime p such that gcd(p, n) = 1, p | x y and
none of x and y are divisible by p, we have
v
p
(x
n
y
n
) = v
p
(x y)
So it suces to show that
v
2
(x
2
n
y
2
n
) = v
2
(x y) + n.
Factorization gives
x
2
n
y
2
n
= (x
2
n1
+ y
2
n1
)(x
2
n2
+ y
2
n2
) (x
2
+ y
2
)(x + y)(x y)
Now since x y 1 (mod 4) then we have x
2
k
y
2
k
1 (mod 4) for all
positive integers k and so x
2
k
+ y
2
k
2 (mod 4), k = 1, 2, 3, . . . . This means
the power of 2 in all of the factors in the above production (except xy) is one.
We are done.
Theorem 4. Let x and y be two odd integers and let n be an even positive
integer. Then
v
2
(x
n
y
n
) = v
2
(x y) + v
2
(x + y) + v
2
(n) 1.
4
Proof. We know that the square of an odd integer is of the form 4k + 1. So
for odd x and y we have 4 | x
2
y
2
. Now let m be an odd integer and k be a
positive integer such that n = m 2
k
. So
v
2
(x
n
y
n
) = v
2
(x
m2
k
y
m2
k
)
= v
2
((x
2
)
2
k1
(y
2
)
2
k1
)
.
.
.
= v
2
(x
2
y
2
) + k 1
= v
2
(x y) + v
2
(x + y) + v
2
(n) 1.
5 Summary
Let p be a prime number and let x and y be two (not necessary positive) integers
which are not divisible by p. Then:
a) For a positive integer n if
p = 2 and p | x y, then
v
p
(x
n
y
n
) = v
p
(x y) + v
p
(n).
p = 2 and 4 | x y, then
v
2
(x
n
y
n
) = v
2
(x y) + v
2
(n).
p = 2 and 2 | x y, then
v
2
(x
n
y
n
) = v
2
(x y) + v
2
(x + y) + v
2
(n) 1.
b) For an odd positive integer n, if p | x + y, then
v
p
(x
n
+ y
n
) = v
p
(x + y) + v
p
(n).
c) For a positive integer n with gcd(p, n) = 1, we have
v
p
(x
n
y
n
) = v
p
(x y).
and if n be odd with gcd(p, n) = 1, then we have
v
p
(x
n
+ y
n
) = v
p
(x + y).
Note. The most common mistake in using LTE is when you dont check the
p | x y condition, so always remember to check it. Otherwise your solution
will be completely wrong.
5
6 Solved Problems
Problem 1 (Russia 1996). Find all positive integers n for which there exist
positive integers x, y and k such that gcd(x, y) = 1, k > 1 and 3
n
= x
k
+ y
k
.
Solution. k should be an odd integer (otherwise if k be even, then x
k
and
y
k
are perfect squares, and it is wellknown that for some integers a, b we have
3 | a
2
+b
2
if and only if 3 | a and 3 | b, which is in contradict with gcd(x, y) = 1.).
Suppose that there exists a prime p such that p | x + y. This prime should
be odd. So v
p
(3
n
) = v
p
(x
k
+ y
k
), and using Theorem 2 we have v
p
(3
n
) =
v
p
(x
k
+ y
k
) = v
p
(k) + v
p
(x + y). But p | x + y means that v
p
(x + y) 1 > 0
and so v
p
(3
n
) = v
p
(k) + v
p
(x + y) > 0 and so p | 3
n
. Thus p = 3. This means
x + y = 3
m
for some positive integer m. Note that n = v
3
(k) + m. There are
two cases:
m > 1. We can prove by induction that 3
a
> a + 2 for all integers a > 1,
and so we have v
3
(k) k 2 (why?). Let M = max(x, y) 5. We have
x
k
+ y
k
M
k
= M
..
x+y
2
=
1
2
3
m
M
k1
. .
5
k1
>
1
2
3
m
5
k1
> 3
m
5
k2
3
m+k2
3
m+v3(k)
= 3
n
.
Which is a contradiction.
m = 1. Then x + y = 3 and so x = 1, y = 2 (or x = 2, y = 1). Thus
3
1+v3(k)
= 1 + 2
k
. But note that we have 3
a
> a for all poisitive integers
a, and this means that 3
v3(k)
k. So
1 + 2
k
= 3
v3(k)+1
= 3 3
v3(k)
. .
k
3k = 2
k
+ 1 3k.
And one can check the only solution for k which satises the above in-
equality is k = 3. And so (x, y, n, k) = (1, 2, 2, 3), (2, 1, 2, 3) in this case.
And the nal solution is n = 2.
Problem 2 (Balkan 1993). Let p be a prime number and m > 1 be a positive
integer. Show that if for some positive integers x > 1, y > 1 we have
x
p
+ y
p
2
=
_
x + y
2
_
m
,
then m = p.
Solution. One can prove by induction that
x
p
+y
p
2
_
x+y
2
_
p
for all positive
integers p. Now since
x
p
+y
p
2
=
_
x+y
2
_
m
, we should have m p. Let d =
gcd(x, y), so there exist positive integers x, y such that x = dx
1
, y = dy
1
and
such that 2
m1
(x
p
1
+ y
p
1
) = d
mp
(x
1
+ y
1
)
m
. There are two cases:
6
Assume that p is odd. Take any prime divisor q of x
1
+y
1
and let v = v
q
(x
1
+
y
1
). If q is odd, we see that v
q
(x
p
1
+y
p
1
) = v+v
q
(p) and v
q
(d
mp
(x
1
+y
1
)
m
) mv
(because q may also be a factor of d). Thus m 2 and p 2, giving an
immediate contradiction. If q = 2, then m 1 + v mv, so v 1 and
x
1
+ y
1
= 2, i.e., x = y, which immediately implies m = p.
Assume that p = 2. We notice that for x+y 4 we have
x
2
+y
2
2
< 2
_
x+y
2
_
2
_
x+y
2
_
3
, so m = 2 and we can check that the solutions are (x, y) = (1, 2), (2, 1)
in this case.
Problem 3. Find all positive integers a, b which are greater than 1 and
b
a
|a
b
1.
Solution. Let p be the least prime divisor of b. Let m be the least positive
integer for which p|a
m
1. Then m|b and m|p 1, so any prime divisor of
m divides b and is less than p. Thus, not to run into a contradiction, we
must have m = 1. Now, if p is odd, we have av
p
(b) v
p
(a 1) + v
p
(b), so
a 1 (a 1)v
p
(b) v
p
(a 1), which is impossible. Thus p = 2, b is even, a is
odd and av
2
(b) v
2
(a1) +v
2
(a+1) +v
2
(b) 1 whence a (a1)v
2
(b) +1
v
2
(a 1) + v
2
(a + 1), which is possible only if a = 3, v
2
(b) = 1. Put b = 2B
with odd B and rewrite the condition as 2
3
B
3
|3
2B
1. Let q be the least prime
divisor of B (now, surely, odd). Let n be the least positive integer such that
q|3
n
1. Then n|2B and n|q 1 whence n must be 2 (or B has a smaller prime
divisor), so q|3
2
1 = 8, which is impossible. Thus B = 1 and b = 2.
Problem 4 (IMO ShortList 2007). Find all surjective functions f : N N such
that for every m, n N and every prime p, the number f(m + n) is divisible by
p if and only if f(m) + f(n) is divisible by p.
Solution. Denote the relation p | a p | b as a b for all primes p. Thus
f(a + b) f(a) + f(b), as problem stated.
Lets prove some lemmas:
Lemma 1. For a prime p, we have p | f(a), p | f(b) if and only if p | f (gcd(a, b)).
Proof. We notice that f(kn) f(n) + f((k 1)n) for all positive integers
k, n with k 2 (here, we replaced a with n and b with (k 1)n in the formula
f(a + b) f(a) + f(b)).
So we can prove by induction that p | f(n) implies p | f(kn) for all k N.
Its clear that gcd(a, b) | a, gcd(a, b) | b, in other words, there exists a
1
, b
1
such that a = gcd(a, b)a
1
and b = gcd(a, b)b
1
. So p | f(gcd(a, b)) implies
p | f(a), p | f(b) (because p | f(gcd(a, b)) = p | f(gcd(a, b)a
1
) = f(a), and
p | f(gcd(a, b)) = p | f(gcd(a, b)b
1
) = f(b), since we showed that p | f(n)
implies p | f(kn)). Thus we showed that p | f (gcd(a, b)) implies p | f(a) and
p | f(b).
By Bezouts identity, there exist positive integers x and y such that ax by =
gcd(a, b). Without loss of generality, suppose that ax by = gcd(a, b). Now,
if p | f(a) and p | f(b), then p | f(ax) and p | f(by) (since p | f(n) implies p |
7
f(nk)). Note that f(ax) f(by)+f(axby), thus p | f(axby) = f(gcd(a, b)).
So we showed that p | f(a), p | f(b) implies p | f (gcd(a, b)) and the proof of
this lemma is complete.
Lemma 2. For a prime p, we have f(x) f(y) if and only if x y (mod p).
Proof. Let m
p
be the smallest positive integer n such that p | f(n). We will
now prove that
p | f(n) m
p
|n (1)
In order to prove (p | f(n) = m
p
|n), suppose that n = m
p
s +t where s, t are
non-negative integers with 0 t < m
p
. Now, if t = 0, we are done. So suppose
that t = 0, thus we should have p | f(t), but this is in contradiction with the
minimality of m
p
(note that 0 < t < m
p
gives the contradiction). So t = 0 and
we proved that p | f(n) = m
p
|n.
And for proving (m
p
|n = p | f(n)), we use the facts that p | f(m
p
) and
n = m
p
and thus
p | f(m
p
) = p | f(m
p
) = f(n)
for some N. This completes the proof of (1).
Without loss of generality suppose that y > x. Take a positive integer a > x
such that m
p
| a. Now,
m
p
| y x p | f(y x) p | f(y x + a) p | f(y) + f(a x).
And since p | f(a), we conclude that p | f(x) + f(a x), so
p|f(y) + f(a x) p|f(y) f(x),
and thus f(x) f(y) (mod p) x y (mod m
p
). This means there exists
a bijection between the residue classes of Z
p
and Z
mp
, so p = m
p
, as desired.
Lemma 3. We have f(p) = p for all primes p of the form 4k + 1 with k N.
Proof. In Lemma 2 we showed that p = m
p
and from (1) we can see that
f(n) n for all n N. So f(1) = 1 and f(p) = p
a
for some a N. This means
m + n f(m + n) f(m) + f(n).
Put m = p and n = 1 in the relation above, we get p + 1 p
a
+ 1. Now since
p 1 (mod 4), we get p + 1 p
a
+ 1 2 (mod 4), so 2p + 1 and 2p
a
+ 1.
Let r = 2 be a prime such that r|p +1, so r|p
a
+1 and this implies r|(1)
a
+1
(if you dont understand this part, write the equation in modulos forms), which
means a should be odd.
From Theorem 2 we see that for every prime r = 2 with r | p + 1, we have
v
r
(p
a
+ 1) = v
r
(p + 1) + v
r
(a). Now,
p
a
+ 1 = 2
r|p+1,r=2
r
vr(p
a
+1)
= 2
r|p+1,r=2
r
vr(p+1)
r
vr(a)
(p + 1)a (2)
8
(we have that 2 behind the product signs because 2p + 1 and 2p
a
+ 1). Try
yourself to prove by induction that p
a
+ 1 (p + 1)a, and the equality occurs
only for a = 1. But in (2) we see that p
a
+1 (p +1)a, thus p
a
+1 = (p +1)a
and so a = 1. Hence the prrof of this lemma is complete.
Now that we have proven these three lemmas, we start proving the problem.
Its clear that x x+1 (mod p) for all x N and all primes p, so using Lemma
2 we have f(x) f(x + 1) (mod p). This means f(x + 1) f(x) = 1 for all
x N. Let q be a prime number of the form 4k + 1, we have
f(q) f(q 1) = 1 1
f(q 1) f(q 2) = 1 1
.
.
.
f(2) f(1) = 1 1
_
_
add up
f(q) f(1) q 1
and equality holds if and only if f(x + 1) f(x) = 1 for all 1 x q 1. But
in Lemma 3 we showed that f(q) = q and f(1) = 1 for all primes q of the form
4k + 1, and thus f(q) f(1) = q 1. This means for a prime q of the form
4k + 1 we have
f(x + 1) f(x) = 1 for all 1 x q 1,
and ince there are arbitrarily large primes of the form 4k + 1, we have f(x +
1) f(x) = 1 for all x N and so
f(x + 1) f(x) = 1
f(x) f(x 1) = 1
.
.
.
f(2) f(1) = 1
_
_
add up
f(x + 1) = x + 1.
Hence f(x) = x for all positive integers x.
9
7 Unsolved Problems
1. Let k be a positive integer. Find all positive integers n such that 3
k
| 2
n
1.
2 (UNESCO Competition 1995). Let a, n be two positive integers and let p be
an odd prime number such that
a
p
1 (mod p
n
).
Prove that
a 1 (mod p
n1
).
3 (Iran Second Round 2008). Show that the only positive integer value of a for
which 4(a
n
+ 1) is a perfect cube for all positive integers n, is 1.
4. Let k > 1 be an integer. Show that there exists innitely many positive
integers n such that
n|1
n
+ 2
n
+ 3
n
+ + k
n
.
5 (Ireland 1996). Let p be a prime number, and a and n positive integers. Prove
that if
2
p
+ 3
p
= a
n
then n = 1.
6 (Russia 1996). Let x, y, p, n, k be positive integers such that n is odd and p
is an odd prime. Prove that if x
n
+ y
n
= p
k
, then n is a power of p.
7. Find the sum of all the divisors d of N = 19
88
1 which are of the form
d = 2
a
3
b
with a, b N.
8. Let p be a prime number. Solve the equation a
p
1 = p
k
in the set of positive
integers.
9. Find all solutions of the equation
(n 1)! + 1 = n
m
in positive integers.
10 (Bulgaria 1997). For some positive integer n, the number 3
n
2
n
is a perfect
power of a prime. Prove that n is a prime.
11. Let m, n, b be three positive integers with m = n and b > 1. Show that if
prime divisors of the numbers b
n
1 and b
m
1 be the same, then b + 1 is a
perfect power of 2.
12 (IMO ShortList 1991). Find the highest degree k of 1991 for which 1991
k
divides the number
1990
1991
1992
+ 1992
1991
1990
.
13. Prove that the number a
a1
1 is never square-free for all integers a > 2.
10
14 (Czech Slovakia 1996). Find all positive integers x, y such that p
x
y
p
= 1,
where p is a prime.
15. Let x and y be two positive rational numbers such that for innitely many
positive integers n, the number x
n
y
n
is a positive integer. Show that x and
y are both positive integers.
16 (IMO 2000). Does there exist a positive integer n such that n has exactly
2000 prime divisors and n divides 2
n
+ 1?
17 (China Western Mathematical Olympiad 2010). Suppose that m and k are
non-negative integers, and p = 2
2
m
+ 1 is a prime number. Prove that
2
2
m+1
p
k
1 (mod p
k+1
);
2
m+1
p
k
is the smallest positive integer n satisfying the congruence equa-
tion 2
n
1 (mod p
k+1
).
18. Let p 5 be a prime. Find the maximum value of positive integer k such
that
p
k
|(p 2)
2(p1)
(p 4)
p1
.
19. Let a, b be distinct real numbers such that the numbers
a b, a
2
b
2
, a
3
b
3
, . . .
Are all integers. Prove that a, b are both integers.
20 (MOSP 2001). Find all quadruples of positive integers (x, r, p, n) such that
p is a prime number, n, r > 1 and x
r
1 = p
n
.
21 (China TST 2009). Let a > b > 1 be positive integers and b be an odd
number, let n be a positive integer. If b
n
| a
n
1, then show that a
b
>
3
n
n
.
22 (Romanian Junior Balkan TST 2008). Let p be a prime number, p = 3,
and integers a, b such that p | a + b and p
2
| a
3
+ b
3
. Prove that p
2
| a + b or
p
3
| a
3
+ b
3
.
23. Let m and n be positive integers. Prove that for each odd positive integer b
there are innitely many primes p such that p
n
1 (mod b
m
) implies b
m1
| n.
24 (IMO 1990). Determine all integers n > 1 such that
2
n
+ 1
n
2
is an integer.
25. Find all positive integers n such that
2
n1
+ 1
n
.
is an integer.
11
26. Find all primes p, q such that
(5
p
2
p
)(5
q
2
q
)
pq
is an integer.
27. For some natural number n let a be the greatest natural nubmer for which
5
n
3
n
is divisible by 2
a
. Also let b be the greatest natural number such that
2
b
n. Prove that a b + 3.
28. Determine all sets of non-negative integers x, y and z which satisfy the
equation
2
x
+ 3
y
= z
2
.
29. Find all positive integer solutions of equation x
2009
+ y
2009
= 7
z
30 (Romania TST 1994). Let n be an odd positive integer. Prove that ((n
1)
n
+ 1)
2
divides n(n 1)
(n1)
n
+1
+ n.
31. Find all positive integers n such that 3
n
1 is divisible by 2
n
.
32 (Romania TST 2009). Let a, n 2 be two integers, which have the following
property: there exists an integer k 2, such that n divides (a 1)
k
. Prove that
n also divides a
n1
+ a
n2
+ + a + 1.
33. Find all the positive integers a such that
5
a
+1
3
a
is a positive integer.
12
8 Hints and Answers to Unsolved Problems
1. Answer: n = 2 3
k1
s for some s N.
2. Show that v
p
(a 1) = v
p
(a
p
1) 1 n 1.
3. If a > 1, a
2
+1 is not a power of 2 (because it is > 2 and either 1 or 2 modulo
4). Choose some odd prime p|a
2
+ 1. Now, take some n = 2m with odd m and
notice that v
p
(4(a
n
+ 1)) = v
p
(a
2
+ 1) + v
p
(m) but v
p
(m) can be anything we
want modulo 3.
5. 2
p
+ 3
p
is not a squre, and use the fact that v
5
(2
p
+ 3
p
) = 1 + v
5
(p) 2.
8. Assume two cases : p = 2 and p be an odd prime. The latter does not have
any solutions.
9. (n, m) = (2, 1) is a solution. In other cases, show that n is an odd prime and
m is even. The other solution is (n, m) = (5, 2).
12. Answer: max(k) = 1991.
13. Take any odd prime p such that p | a 1. Its clear that p
2
| a
a1
1.
14. Answer: (p, x, y) = (2, 1, 1), (3, 2, 1).
18. Let p 1 = 2
s
m and show that v
p
(2
s1
m) = 0. The maximum of k is 1.
19. Try to prove Problem 15 rst.
20. Show that p = 2 and r is an even positive integer.
22. If p | a, p | b, then p
3
| a
3
+ b
3
. Otherwise LTE applies and v
p
(a + b) =
v
p
(a
3
+ b
3
) 2.
24. The answer is n = 1 or n = 3.
26. Answer: (p, q) = (3, 3), (3, 13).
27. If n is odd, then a = 1. If n is even, then a = v
2
(5
n
3
n
) = v
2
(5 3) +
v
2
(5 + 3) + v
2
(n) 1 = 3 + v
2
(n). But, clearly, b v
2
(n).
30. n | (n 1)
n
+ 1, so for every p | (n 1)
n
+ 1, we have
v
p
((n 1)
(n1)
n
+1
+ 1) = v
p
((n 1)
n
+ 1) + v
p
_
(n 1)
n+1
+ 1
n
_
= 2v
p
((n 1)
n
+ 1) v
p
(n)
which completes the proof.
31. n v
2
(3
n
1) 3 + v
2
(n), so n 4.
33. a must be odd (otherwise the numerator is 2 mod 3). Then a v
3
(5
a
+1) =
1 + v
3
(a) giving a = 1 as the only solution.
13
References
[1] Sepehr Ghazi Nezami, Leme Do Khat (in English: Lifting The Exponent
Lemma) published on October 2009.
http://imo09.blogfa.com/page/2khat.aspx
[2] AoPS topic #393335, Lifting The Exponent Lemma (Containing PDF
le), posted by amparvardi.
http://www.artofproblemsolving.com/Forum/viewtopic.php?t=393335
[3] Santiago Cuellar, Jose Alejandro Samper, A nice and tricky lemma (lift-
ing the exponent), Mathematical Reections 3 - 2007.
[4] AoPS topic #214717, Number mod (f(m+n), p) = 0 i mod (f(m) +
f(n), p) = 0, posted by orl.
http://www.artofproblemsolving.com/Forum/viewtopic.php?t=214717
[5] AoPS topic #268964, China TST, Quiz 6, Problem 1, posted by Fang-jh.
http://www.artofproblemsolving.com/Forum/viewtopic.php?t=268964
[6] AoPS topic #57607, exactly 2000 prime divisors, posted by Valentin Vor-
nicu.
http://www.artofproblemsolving.com/Forum/viewtopic.php?t=57607
[7] AoPS topic #220915, Highest degree for 3-layer power tower, posted by
orl.
http://www.artofproblemsolving.com/Forum/viewtopic.php?t=220915
[8] AoPS topic #368210, Iran NMO 2008 (Second Round) - Problem4, posted
by sororak.
http://www.artofproblemsolving.com/Forum/viewtopic.php?t=368210
14